answer key for niacl ao quant questions€¦ · मजदूरोऻ की सऻख्या...

35
1 | Page Answer Key for NIACL AO Quant Questions Q1. Ans: 2 Solution: The pattern of given series is as: 10 = 10 × 1 - 0 19 = 10 × 2 - 1 28 = 10 × 3 - 2 37 = 10 × 4 - 3 46 = 10 × 5 - 4 = ? Q2. Ans: 4 Solution: Q1 2 Q2 4 Q3 2 Q4 3 Q5 1 Q 6 3 Q 7 2 Q 8 3 Q 9 1 Q 10 2 Q 11 2 Q 12 2 Q 13 1 Q 14 5 Q 15 1 Q 16 5 Q 17 4 Q 18 3 Q 19 3 Q 20 3 Q 21 3 Q 22 4 Q 23 5 Q 24 3 Q 25 5 Q 26 2 Q 27 3 Q 28 3 Q 29 4 Q 30 4 Q 31 4 Q 32 4 Q 33 2 Q 34 1 Q 35 2

Upload: others

Post on 11-Jun-2020

3 views

Category:

Documents


0 download

TRANSCRIPT

  • 1 | P a g e

    Answer Key for NIACL AO Quant

    Questions

    Q1.

    Ans: 2

    Solution:

    The pattern of given series is as:

    ⇒ 10 = 10 × 1 - 0

    ⇒ 19 = 10 × 2 - 1

    ⇒ 28 = 10 × 3 - 2

    ⇒ 37 = 10 × 4 - 3

    ⇒ 46 = 10 × 5 - 4 = ?

    Q2.

    Ans: 4

    Solution:

    Q1 2 Q2 4 Q3 2 Q4 3 Q5 1

    Q 6 3 Q 7 2 Q 8 3 Q 9 1 Q 10 2

    Q 11 2 Q 12 2 Q 13 1 Q 14 5 Q 15 1 Q 16 5 Q 17 4 Q 18 3 Q 19 3 Q 20 3

    Q 21 3 Q 22 4 Q 23 5 Q 24 3 Q 25 5

    Q 26 2 Q 27 3 Q 28 3 Q 29 4 Q 30 4

    Q 31 4 Q 32 4 Q 33 2 Q 34 1 Q 35 2

    https://testbook.com/passhttps://play.google.com/store/apps/details?id=com.testbook.tbapp&hl=en

  • 2 | P a g e

    The given number series is based on the following pattern.

    18 × 1 + 2 = 20

    20 × 2 + 4 = 44

    44 × 3 + 6 = 138

    138 × 4 + 8 = 560

    560 × 5 + 10 = 2810

    ? = 2810 × 6 + 12 = 16872

    Q3.

    Ans: 2

    Solution:

    The given number series is based on the following pattern.

    29 × 29 = 841

    31 × 31 = 961

    33 × 33 = 1089

    35 × 35 = 1225

    37 × 37 = 1369

    39 × 39 = 1521

    ? = 41 × 41 = 1681

    Q4.

    Ans: 3

    Solution:

    https://testbook.com/passfile:///C:/Users/Administrator/Downloads/www.bit.ly/bank-live

  • 3 | P a g e

    The given number series is based on the following pattern.

    963 - 1 × 36 = 927

    927 - 2 × 36 = 855

    747 - 4 × 36 = 603

    603 - 5 × 36 = 423

    ? = 423 - 6 × 36 = 207

    Q5.

    Ans: 1

    Solution:

    The given number series is based on the following pattern.

    1200 ÷ 2.5 = 480

    480 ÷ 2.5 = 192

    192 ÷ 2.5 = 76.8

    76.8 ÷ 2.5 = 30.72

    30.72 ÷ 2.5 = 12.288

    ? = 12.288 ÷ 2.5 = 4.9152

    Q6.

    Ans: 3

    Solution:

    2? = 32.01 ÷ 128.01 × 1023.97 ÷ 7.97

    By approximation;

    https://testbook.com/passhttps://play.google.com/store/apps/details?id=com.testbook.tbapp&hl=en

  • 4 | P a g e

    ⇒ 2? = 32 ÷ 128 × 1024 ÷ 8 = 32 ÷ 128 × 128

    2? = 32 = 25

    ⇒ ? = 5

    2? = 32.01 ÷ 128.01 × 1023.97 ÷ 7.97

    अनुमाननकरण

    ⇒ 2? = 32 ÷ 128 × 1024 ÷ 8 = 32 ÷ 128 × 128

    2? = 32 = 25

    ⇒ ? = 5

    Q7.

    Ans: 2

    Solution:

    339.98 ÷ ? = √143.98 + √64.02

    By approximation;

    ⇒ 340 ÷ ? = √144 + √64 = 20

    ⇒ ? = 340/20 = 17

    339.98 ÷ ? = √143.98 + √64.02

    अनुमाननकरण

    ⇒ 340 ÷ ? = √144 + √64 = 20

    ⇒ ? = 340/20 = 17

    https://testbook.com/passfile:///C:/Users/Administrator/Downloads/www.bit.ly/bank-live

  • 5 | P a g e

    Q8.

    Ans: 3

    Solution:

    34.03% of 550.08 ÷ ? = 297.08 + √728.97 - √89998

    By approximation;

    ⇒ 34% of 550 ÷ ? = 297 + √729 - √90000

    187 ÷ ? = 297 + 27 - 300 = 24

    ⇒ ? = 8 (approx.)

    34.03% of 550.08 ÷ ? = 297.08 + √728.97 - √89998

    अनुमाननकरण

    ⇒ 34% of 550 ÷ ? = 297 + √729 - √90000

    187 ÷ ? = 297 + 27 - 300 = 24

    ⇒ ? = 8 (अनुमाननत)

    Q9.

    Ans: 1

    Solution:

    (? ÷ 9.97) × 12.08 = 20.12% of 1319.98

    By approximation;

    https://testbook.com/passhttps://play.google.com/store/apps/details?id=com.testbook.tbapp&hl=en

  • 6 | P a g e

    (? ÷ 10) × 12 = 20% of 1320

    ? ÷ 10 × 12 = 264

    ⇒ ? = 264 × 10/12 = 220

    (? ÷ 9.97) × 12.08 = 20.12% of 1319.98

    अनुमाननकरण

    (? ÷ 10) × 12 = 20% of 1320

    ? ÷ 10 × 12 = 264

    ⇒ ? = 264 × 10/12 = 220

    Q10.

    Ans: 2

    Solution:

    ?% of 179.98 = √((24.03)2 + (17.99)2 + (60.02% of 659.96))

    By approximation;

    ?% of 180 = √((24)2 + (18)2 + (60% of 660))

    ?% of 180 = √(576 + 324 + 396) = √1296 = 36

    ⇒ ?% = 36/180 = 1/5

    ⇒ ? = 20

    179.98 का ?% = √((24.03)2 + (17.99)2 + (659.96 का 60.02%))

    अनुमाननकरण द्वारा;

    180 का ?% = √((24)2 + (18)2 + (660 का 60%))

    https://testbook.com/passfile:///C:/Users/Administrator/Downloads/www.bit.ly/bank-live

  • 7 | P a g e

    180 का ?% = √(576 + 324 + 396) = √1296 = 36

    ⇒ ?% = 36/180 = 1/5

    ⇒ ? = 20

    Q11.

    Ans: 2

    Solution:

    Quantity A:

    Probability of getting green ball is 1/4

    We know that, Probability = Favourable outcomes/Total outcomes

    Probability = x/(12 + x)

    1/4 = x/(12 + x)

    3x = 12

    x = 4

    Quantity B = 4

    Hence Quantity A = Quantity B

    मात्रा A:

    हरे गेंद को प्राप्त करने की प्रानिकता 1/4 है।

    हम जानते हैं नक, अनुकूल पररणाम /कुल पररणाम

    प्रानिकता = x/(12 + x)

    1/4 = x/(12 + x)

    https://testbook.com/passhttps://play.google.com/store/apps/details?id=com.testbook.tbapp&hl=en

  • 8 | P a g e

    3x = 12

    x = 4

    मात्रा B = 4

    अतः मात्रा A = मात्रा B

    Q12.

    Ans: 2

    Solution:

    First we will find Quantity A,

    Quantity A:

    Here, Cost price (CP) = 140 and

    Selling Price (SP) = 168

    % Profit = (SP – CP)/CP × 100

    ⇒ % Profit = (168 – 140)/140 × 100 = 20

    Now, Quantity B:

    Here, Marked Price (MP) = 2000

    Selling Price (SP) = 1800

    % Discount = (MP – SP)/MP × 100

    ⇒ % Discount = (2000 – 1800)/2000 × 100 = 10

    Clearly, Quantity B < Quantity A

    पहले हमें मात्रा A को ज्ञात करना होगा,

    मात्रा A:

    िहााँ, क्रि मूल्य (CP) = 140

    https://testbook.com/passfile:///C:/Users/Administrator/Downloads/www.bit.ly/bank-live

  • 9 | P a g e

    निक्रि मूल्य (SP) = 168

    % लाभ = (SP – CP)/CP × 100

    ⇒ % लाभ = (168 – 140)/140 × 100 = 20

    अब,

    मात्रा B:

    िहााँ, अंनकत मूल्य (MP) = 2000

    निक्रि मूल्य (SP) = 1800

    प्रनतशत छूट = (MP – SP)/MP × 100

    ⇒ % छूट = (2000 – 1800)/2000 × 100 = 10

    स्पष्ट रूप से, मात्रा B < मात्रा A

    Q13.

    Ans: 1

    Solution:

    Firstly we will find Quantity A,

    Quantity A:

    Number of ways to choose 2 students from 5 students = 5C2 = 10

    Number of ways to choose 2 teachers from 3 teachers = 3C2 = 3

    Number of ways to choose 1 coach from 2 coaches = 2C1 = 2

    ∴ Total number of ways = 10 × 3 × 2 = 60

    Now,

    Quantity B:

    https://testbook.com/passhttps://play.google.com/store/apps/details?id=com.testbook.tbapp&hl=en

  • 10 | P a g e

    Total number of people = 5 + 3 + 2 = 10

    Number of ways to choose 5 people from 10 people = 10C5 = 252

    Quantity B > Quantity A

    पहले हमें मात्रा A को ज्ञात करना होगा,

    मात्रा A:

    5 छात्रो ंमें से 2 छात्रो ंको चुनने के प्रकार = 5C2 = 10

    3 नशक्षको ंमें से 2 नशक्षको ंको चुनने के प्रकार = 3C2 = 3

    2 प्रनशक्षको ंमें से 1 प्रनशक्षक चुनने के प्रकार = 2C1 = 2

    ∴ प्रकारो ंकी कुल संख्या = 10 × 3 × 2 = 60

    अब,

    मात्रा B:

    व्यक्तििो ंकी कुल संख्या = 5 + 3 + 2 = 10

    10 व्यक्तििो ंमें से 5 व्यक्तििो ंको चुनने के प्रकार = 10C5 = 252

    मात्रा B > मात्रा A

    Q14.

    Ans: 5

    Solution:

    We know that LCM is always a multiple of HCF.

    Let LCM = k(HCF)

    https://testbook.com/passfile:///C:/Users/Administrator/Downloads/www.bit.ly/bank-live

  • 11 | P a g e

    ⇒ HCF + k(HCF) = 37

    ⇒ (k + 1)(HCF) = 37

    Possible values of k are 0 and 36.

    If k = 0, HCF = 37, which is not possible as LCM will become 0.

    If k = 36, HCF = 1, and LCM = 36.

    So, P and Q can be 1 and 36, or 4 and 9.

    If they are 1 and 36, their sum is 37 and product is 36.

    If they are 4 and 9, their sum is 13 and product is 36.

    So, sum can be lesser or greater than product.

    No relation between A and B can be established.

    हम जानते हैं नक ल.स.प. हमेंशा म.स.प. का भाज्य होता है।

    माना नक ल.स.प. = k(म.स.प.)

    ⇒ म.स.प. + k(म.स.प.) = 37

    ⇒ (k + 1)(HCF) = 37

    K का संभि मान 0 और 36 है।

    िनद k = 0, HCF = 37, जो संभि नही ंहै क्ोनंक ल.स.प. 0 हो जािेगा।

    िनद k = 36, HCF = 1, और ल.स.प. = 36.

    अत: P और Q ,1 और 36, िा 4 और 9 हो सकते हैं।

    िनद िे 1 और 36 हैं, तो इनका िोग 37 और गुणनफल 36 होगा।

    िनद िे 4 और 9 हैं, तो इनका िोग 13 और गुणनफल 36 होगा।

    अत: िोग गुणनफल से कम िा अनिक हो सकता है।

    https://testbook.com/passhttps://play.google.com/store/apps/details?id=com.testbook.tbapp&hl=en

  • 12 | P a g e

    A और B के बीच में कोई संबंि स्थानपत नही ंनकिा जा सकता।

    Q15.

    Ans: 1

    Solution:

    First we will find Quantity A,

    Quantity A:

    No. of hours taken to complete a home work = 2 hrs

    Total no. of hours he worked = 6 hrs

    ∴ No.of homeworks he can complete =( Total no. of hours worked)/(No. of hours taken to

    complete each home work) = 6/2 = 3 home-works

    Now,

    Quantity B:

    Work done by A in 1 day = 1/20

    Work done by B in 1 day = 1/25

    Work done by C in 1 day = 1/10

    Work done by A & B together in 9 days = 9 × (1

    20+

    1

    25) =

    81

    100

    Amount of work remaining = 1 −81

    100=

    19

    100

    Work to be done by A, B & C in 'x' days = 19/100

    ⇒ 𝑥 × (1

    20+

    1

    25+

    1

    10) =

    19

    100

    ⇒ x = 1 day

    ⇒ No. of days in which the remaining work gets completed = 1

    https://testbook.com/passfile:///C:/Users/Administrator/Downloads/www.bit.ly/bank-live

  • 13 | P a g e

    ∴ Quantity A > Quantity B

    पहले हमें मात्रा A ज्ञात करना होगा,

    मात्रा A:

    गृहकािय को पूरा करने के नलए नलिा गिा समि = 2 घंटे

    उसके कुल घंटे तक काम नकिा = 6 घंटे

    ∴ उसके द्वारा पूरे नकिे गिे गृहकािय की संख्या = (काम नकिे गिे कुल घंटे)/(प्रते्यक द्वारा गृहकािय को पूरा

    करने के नलए नलिे गिे घंटो ंकी संख्या) = 6/2 = 3 गृहकािय

    अब,

    मात्रा B:

    1 नदन में A द्वारा नकिा गिा काम = 1/20

    1 नदन में B द्वारा नकिा गिा काम = 1/25

    1 नदन में C द्वारा नकिा गिा काम = 1/10

    9 नदनो ंमें A और B दोनो ंद्वारा नकिा गिा काम = 9 × (1

    20+

    1

    25) =

    81

    100

    काम का शेष बचा भाग = 1 −81

    100=

    19

    100

    'x' नदनो ंमें A, B और C द्वारा काम होगा = 19/100

    ⇒ 𝑥 × (1

    20+

    1

    25+

    1

    10) =

    19

    100

    ⇒ x = 1 नदन

    ⇒ शेष बचे काम को पूरा करने के नलए नदनो ंकी संख्या = 1 नदन

    ∴ मात्रा A > मात्रा B

    https://testbook.com/passhttps://play.google.com/store/apps/details?id=com.testbook.tbapp&hl=en

  • 14 | P a g e

    Comprehension Start

    Q16.

    Ans: 5

    Solution:

    Number of workers = (Percentage of workers) × (Total Workers)

    Number of officers = (Ratio of officers : workers) × (number of workers)

    Company Percentage of workers

    Number of workers Officers: Workers Number of officers

    A 32% 288 1 : 16 18

    B 44% 396 1 : 18 22

    C 24% 216 1 : 12 18

    Number of officers in A and B together = 18 + 22 = 40

    Number of officers in B and C together = 22 + 18 = 40

    Thus, the ratio of officers of company A and B together to the officers of company B and C together = 1 : 1

    मजदूरो ंकी संख्या = (मजदूरो ंका प्रनतशत) × (कुल मजदूर)

    ऑनफसरो ंकी संख्या = (ऑनफसरो ं: मजदूरो ंका अनुपात) × (मजदूरो ंकी संख्या)

    कंपनी मजदूरो ंका

    प्रनतशत मजदूरो ंकी संख्या ऑनफसर : मजदूर ऑनफसरो ंकी संख्या

    A 32% 288 1 : 16 18

    B 44% 396 1 : 18 22

    C 24% 216 1 : 12 18

    https://testbook.com/passfile:///C:/Users/Administrator/Downloads/www.bit.ly/bank-live

  • 15 | P a g e

    A और B में ऑनफसरो ंकी संख्या = 18 + 22 = 40

    B और C में ऑनफसरो ंकी संख्या = 22 + 18 = 40

    अतः कंपनी A और B के ऑनफसरो ंतथा कंपनी B और C के ऑनफसरो ंका अनुपात = 1 : 1

    Q17.

    Ans: 4

    Solution:

    Company Percentage of workers

    Number of workers Officers : Workers Number of

    officers Total

    employees

    B 44% 396 1 : 18 22 418

    C 24% 216 1 : 12 18 234

    Percentage difference = ((final - initial)/initial) × 100%

    = ((418 - 234)/234) × 100% = 78.63% ≈ 79%

    कंपनी मजदूरो ंका

    प्रनतशत मजदूरो ंकी संख्या ऑनफसर : मजदूर

    ऑनफसरो ंकी

    संख्या कुल कमयचारी

    B 44% 396 1 : 18 22 418

    C 24% 216 1 : 12 18 234

    प्रनतशत अंतर = ((अंनतम - प्रारंनभक)/प्रारंनभक) × 100%

    = ((418 - 234)/234) × 100% = 78.63% ≈ 79%

    Q18

    https://testbook.com/passhttps://play.google.com/store/apps/details?id=com.testbook.tbapp&hl=en

  • 16 | P a g e

    Ans: 3

    Solution:

    Company Number of workers Number of officers Total employees

    A 288 18 306

    B 396 22 418

    C 216 18 234

    total 900 58 958

    Percentage of officers of company A = 18

    958× 100% = 1.88%

    Converting it to degrees,

    1.88% of 360° = 6.76°

    कंपनी मजदूरो ंकी संख्या ऑनफसरो ंकी संख्या कुल कमयचारी

    A 288 18 306

    B 396 22 418

    C 216 18 234

    कुल 900 58 958

    कंपनी A के ऑनफसरो ंका प्रनतशत = 18

    958×100% = 1.88%

    इसे निग्री में बदलने पर,

    360° का 1.88% = 6.76°

    Q19.

    Ans: 3

    https://testbook.com/passfile:///C:/Users/Administrator/Downloads/www.bit.ly/bank-live

  • 17 | P a g e

    Solution:

    Number of workers of company C = 24% of 900

    Number of workers of company B = 44% of 900

    Ratio of number of workers of company C to that of company B = 24% of 900 : 44% of 900 = 24 : 44 = 6 : 11

    कंपनी C के मजदूरो ंकी संख्या = 900 का 24%

    कंपनी B के मजदूरो ंकी संख्या = 900 का 44%

    कंपनी C के मजदूरो ंकी संख्या और कंपनी B के मजदूरो ंकी संख्या का अनुपात = 900 का 24% : 900 का 44%= 24 : 44 = 6 : 11

    Q20.

    Ans: 3

    Solution:

    Company Number of workers Number of officers

    A 288 18

    B 396 22

    C 216 18

    Officers of company A and C together = 18 + 18 = 36

    Workers of company B = 396

    Ratio of officers of company A and C together workers of company B = 36 : 396 = 1 : 11

    कंपनी मजदूरो ंकी संख्या ऑनफसरो ंकी संख्या

    https://testbook.com/passhttps://play.google.com/store/apps/details?id=com.testbook.tbapp&hl=en

  • 18 | P a g e

    A 288 18

    B 396 22

    C 216 18

    कंपनी A और C के ऑनफसर = 18 + 18 = 36

    कंपनी B के मजदूर = 396

    कंपनी A और C के कुल ऑनफसरो ंऔर कंपनी B के मजदूरो ंका अनुपात =

    36 : 396 = 1 : 11

    Q21.

    Ans: 3

    Solution:

    In third year,

    A’s investment = Rs. 50000 for 12 months

    B’s investment = Rs. 40000 for 12 months

    C’s investment = Rs. 60000 for 12 months

    Hence,

    ⇒ A’s share : B’s share : C’s share = 50000 : 40000 : 60000

    ⇒ A’s share : B’s share : C’s share = 5 : 4 : 6

    ∴ C’s share = (6/15) × 60000 = Rs. 24000

    तीसरे िषय में,

    A का ननिेश = 12 महीनो ंके नलए 50000 रुपिे

    https://testbook.com/passfile:///C:/Users/Administrator/Downloads/www.bit.ly/bank-live

  • 19 | P a g e

    B का ननिेश = 12 महीनो ंके नलए 40000 रुपिे

    C का ननिेश = 12 महीनो ंके नलए 60000 रुपिे

    अतः,

    ⇒ A का नहस्सा : B का नहस्सा : C का नहस्सा = 50000 : 40000 : 60000

    ⇒ A का नहस्सा : B का नहस्सा : C का नहस्सा = 5 : 4 : 6

    ∴ C का नहस्सा = (6/15) × 60000 = 24000 रुपिे

    Q22.

    Ans: 4

    Solution:

    In second year,

    A’s investment = Rs. 50000 for 12 months

    C’s investment = Rs. 60000 for (12 – 3) = 9 months

    C’s share = Rs. 4500

    Hence,

    ⇒ (50000 × 12) : (60000 × 9) = A’s share : 4500

    ⇒ 10 : 9 = A’s share : 4500

    ⇒ A’s share = (10/9) × 4500

    ∴ A’ share = Rs. 5000

    दूसरे िषय में,

    A का ननिेश = 12 महीनो ंके नलए 50000 रुपिे

    C का ननिेश = (12 – 3) = 9 महीनो ंके नलए 60000 रुपिे

    https://testbook.com/passhttps://play.google.com/store/apps/details?id=com.testbook.tbapp&hl=en

  • 20 | P a g e

    C का नहस्सा = 4500 रुपिे

    अतः,

    ⇒ (50000 × 12) : (60000 × 9) = A का नहस्सा : 4500

    ⇒ 10 : 9 = A का नहस्सा : 4500

    ⇒ A का नहस्सा = (10/9) × 4500

    ∴ A का नहस्सा = 5000 रुपिे

    Q23.

    Ans: 5

    Solution:

    A’s investment = Rs. 50000 for 12 months

    B’s investment = Rs. 40000 for (12 – 4) = 8 months

    ∴ Ratio of their share in profit = (50000 × 12) : (40000 × 8) = 15 : 8

    A का ननिेश = 12 महीनो ंके नलए 50000 रुपिे

    B का ननिेश = (12 – 4) = 8 महीनो ंके नलए 40000 रुपिे

    ∴ उनके लाभांश का अनुपात = (50000 × 12) : (40000 × 8) = 15 : 8

    Q24.

    Ans: 3

    Solution:

    Let B invested in the company after ‘x’ months

    https://testbook.com/passfile:///C:/Users/Administrator/Downloads/www.bit.ly/bank-live

  • 21 | P a g e

    A’s investment = Rs. 50000 for 12 months

    B’s investment = Rs. 40000 for (12 – x) months

    A’s share = Rs. 12000

    B’s share = Rs. 5600

    Hence,

    ⇒ (50000 × 12) : [40000 × (12 – x) ] = 12000 : 5600

    ⇒ (5 × 12) : [4 × (12 – x) ] = 120 : 56

    ⇒ 60 : (48 – 4x) = 120 : 56

    ⇒ (60 × 56)/120 = 48 – 4x

    ⇒ 28 = 48 – 4x

    ⇒ 4x = 20

    ⇒ x = 5

    ∴ B invested in the company after 5 months

    माना B ने कंपनी में ‘x’ महीनो ंके बाद ननिेश नकिा

    A का ननिेश = 12 महीनो ंके नलए 50000 रुपिे

    B का ननिेश = (12 – x) महीनो ंके नलए 40000 रुपिे

    A का नहस्सा = 12000 रुपिे

    B का नहस्सा = 5600 रुपिे

    अतः,

    ⇒ (50000 × 12) : [40000 × (12 – x) ] = 12000 : 5600

    https://testbook.com/passhttps://play.google.com/store/apps/details?id=com.testbook.tbapp&hl=en

  • 22 | P a g e

    ⇒ (5 × 12) : [4 × (12 – x) ] = 120 : 56

    ⇒ 60 : (48 – 4x) = 120 : 56

    ⇒ (60 × 56)/120 = 48 – 4x

    ⇒ 28 = 48 – 4x

    ⇒ 4x = 20

    ⇒ x = 5

    ∴ B ने कंपनी में 5 महीनो ंके बाद ननिेश नकिा।

    Q25.

    Ans: 5

    Solution:

    As the time duration for which B invested his money is not known, his share of profit couldn’t

    be determined.

    िह समि अिनि नजसके नलए B ने अपनी रानश का ननिेश नकिा िह ज्ञात नही ंहै, इसनलए उसका लाभांश

    ज्ञात नही ंनकिा जा सकता है।

    Q26.

    Ans: 2

    Solution:

    Let the total work be LCM(10, 12, 20) = 60 units

    ⇒ Efficiency of A = 60/10 = 6 units/day

    ⇒ Efficiency of B = 60/12 = 5 units/day

    https://testbook.com/passfile:///C:/Users/Administrator/Downloads/www.bit.ly/bank-live

  • 23 | P a g e

    ⇒ Efficiency of C = 60/20 = 3 units/day

    Since the number of working days are different for each person, the share of each will be calculated in the ratio of the units of work done

    Now, A works for 2 days and B works for 3 days

    ⇒ Work done by A = 2 x 6 = 12 units

    ⇒ Work done by B = 3 x 5 = 15 units

    ⇒ Work done by C = 60 – 12 – 15 = 33 units

    Therefore, ratio of work done = 12 : 15 : 33 = 4 : 5 : 11

    ⇒ A’s share = (4/20) x 2,00,000 = Rs 40,000

    ⇒ B’s share = (5/20) x 2,00,000 = Rs 50,000

    ⇒ C’s share = (11/20) x 2,00,000 = Rs 1,10,000

    Difference of the highest and lowest share = Rs 1,10,000 – 40,000 = Rs 70,000

    ∴ Difference is of Rs. 70000

    माना कुल काम (10, 12, 20) का लघुत्तम समापित्यय अथायत् 60 इकाई है।

    ⇒ A की काियक्षमता = 60/10 = 6 इकाई/नदन

    ⇒ B की काियक्षमता = 60/12 = 5 इकाई/नदन

    ⇒ C की काियक्षमता = 60/20 = 3 इकाई/नदन

    चंूनक प्रते्यक व्यक्ति के नलए काम के नदनो ंकी संख्या नभन्न है, इसनलिे प्रते्यक का नहस्सा उसके द्वारा नकिे गए

    काम की इकाई के अनुपात में नकिा जाएगा।

    अब, A 2 नदनो ंके नलए काम करता है और B 3 नदनो ंके नलए काम करता है।

    ⇒ A द्वारा नकिा गिा काम = 2 x 6 = 12 इकाई

    ⇒ B द्वारा नकिा गिा काम = 3 x 5 = 15 इकाई

    ⇒ C द्वारा नकिा गिा काम = 60 – 12 – 15 = 33 इकाई

    https://testbook.com/passhttps://play.google.com/store/apps/details?id=com.testbook.tbapp&hl=en

  • 24 | P a g e

    इस प्रकार, नकिे गए काम का अनुपात = 12 : 15 : 33 = 4 : 5 : 11

    ⇒ A का नहस्सा = (4/20) x 2,00,000 = 40,000 रू.

    ⇒ B का नहस्सा = (5/20) x 2,00,000 = 50,000 रू.

    ⇒ C का नहस्सा = (11/20) x 2,00,000 = 1,10,000 रू.

    सबसे अनिक तथा सबसे कम नहसे्स का अंतर = 1,10,000 – 40,000 = 70,000 रू.

    ∴ 70000 रू. का अंतर है।

    Q27.

    Ans: 3

    Solution:

    Outcomes will be different in following cases: HT & TH

    ∴ required probability = 0.7 × (1 – 0.7) + (1 – 0.7) × 0.7 = 0.42

    नभन्न मान इस प्रकार आएगा: नचत पट (HT) और पट नचत (TH)

    ∴संभािना = 0.7 × (1 – 0.7) + (1 – 0.7) × 0.7 = 0.42

    Q28.

    Ans: 3

    Solution:

    If train B enters the tunnel 30 seconds later and train A takes one minute to cross the tunnel, this implies that train B takes 30 seconds to cross the tunnel as both the trains leave the tunnel at the same time.

    https://testbook.com/passfile:///C:/Users/Administrator/Downloads/www.bit.ly/bank-live

  • 25 | P a g e

    Distance covered by train B = length of train + length of the tunnel = 120 + 300 = 420 m

    Thus, speed = (Distance covered)/(Time taken)

    ∴ Speed of train B = 420/30 = 14 m/s

    िनद टर ेन B 30 सेकंि बाद सुरंग में प्रिेश करती है और टर ेन A सुरंग को पार करने के नलए एक नमनट लेती है,

    तो इसका अथय है नक टर ेन B सुरंग को पार करने में 30 सेकंि लगाती हैं क्ोनंक दोनो ंटर ेनें एक ही समि में

    सुरंग से बाहर आती हैं।

    टर ेन B द्वारा ति की गई दूरी = टर ेन की लंबाई + सुरंग की लबाई = 120 + 300 = 420 मीटर

    इसनलए, गनत = (ति की गई दूरी)/(नलिा गिा समि)

    ∴ टर ेन B की गनत = 420/30 = 14 मीटर/सेकंि

    Q29.

    Ans: 4

    Solution:

    Let the height of the cuboid be ‘h’ units

    Length of cuboid = l = 4h units

    Breadth of cuboid = b = 3h units

    Now,

    Diagonal of a cuboid = √(l2 + b2 + h2) = √(16h2 + 9h2 + h2) = √(26h2) = h√26 units

    ∴ The diagonal of the cuboid is √26 times the height of the cuboid

    https://testbook.com/passhttps://play.google.com/store/apps/details?id=com.testbook.tbapp&hl=en

  • 26 | P a g e

    मान लीनजए नक घनाभ की ऊंचाई ‘h' इकाई है

    घनाभ की लंबाई = l = 4h इकाई

    घनाभ की चौडाई = b = 3h इकाई

    अब,

    एक घनाभ का निकणय = √(l2 + b2 + h2) = √(16h2 + 9h2 + h2) = √(26h2) = h√26 इकाई

    ∴ घनाभ का निकणय, घनाभ की ऊंचाई के √26 गुना है

    Q30.

    Ans: 4

    Solution:

    Let the speed of the current be c and the speed of the boat be s Km/hr.

    While going upstream, the effective speed of the boat is s – c Km/hr.

    While going downstream, the effective speed of the boat is s + c Km/hr.

    Time = Distance/Speed

    According to the question

    9/(s – c) + 38/(s + c) = 3 ----(i)

    45/(s – c) + 57/(s + c) = 8 ----(ii)

    To solve these equations, we first multiply (i) by 5 and subtract (ii) from (i) i.e. (i) × 5 – (ii)

    ⇒ 45/(s – c) + 38 × 5/(s + c) – 45/(s – c) – 57/(s + c) = 15 – 8

    ⇒ (5 × 38 – 57)/(s + c) = 7

    ⇒ 133/(s + c) = 7

    ⇒ s + c = 19 ----(iii)

    Put s + c = 19 in eqn(i) we get,

    ⇒ s – c = 9 ----(iv)

    https://testbook.com/passfile:///C:/Users/Administrator/Downloads/www.bit.ly/bank-live

  • 27 | P a g e

    Solving eqn (iii) and eqn (iv) simultaneously we get,

    ⇒ c = 5 Km/hr

    Therefore, the speed of the current is 5 Km/hr

    मान लेते हैं नक प्रिाह की गनत c है एिं नाि की गनत s नकमी/घंटा है

    अनुप्रिाह जाते समि, नाि की प्रभािी गनत s - c नकमी/घंटा है

    प्रनतप्रिाह जाते समि, नाि की प्रभािी गनत s + c नकमी/घंटा है

    समि = दूरी/गनत

    प्रश्न के मुतानबक

    9/(s – c) + 38/(s + c) = 3 ----(i)

    45/(s – c) + 57/(s + c) = 8 ----(ii)

    इन समीकरणो ंका हल ननकलने के नलए, हम पहले (i) को 5 से गुणा करते हैं एिं (i) से (ii) को घटाते हैं

    अथायत् (i) × 5 – (ii)

    ⇒ 45/(s – c) + 38 × 5/(s + c) – 45/(s – c) – 57/(s + c) = 15 – 8

    ⇒ (5 × 38 – 57)/(s + c) = 7

    ⇒ 133/(s + c) = 7

    ⇒ s + c = 19 -----(iii)

    s + c = 19 को समीकरण (i) में िालने से, हमें नमलता है,

    ⇒ s – c = 9 ----(iv)

    एक साथ समीकरण (iii) एिं समीकरण (iv) का हल करने से हमें नमलता है,

    ⇒ c = 5 नकमी/घंटा

    अतः, प्रिाह की गनत 5 नकमी/घंटा है

    Q31.

    https://testbook.com/passhttps://play.google.com/store/apps/details?id=com.testbook.tbapp&hl=en

  • 28 | P a g e

    Ans: 4

    Solution:

    Let the quantity sold at 15% profit be x kg. Then the quantity sold at 5% loss will be (200 –x) kg

    Let us consider price of the rice be Rs.1 per kg

    Then the price of x kg of rice = Rs. x and price of (200 – x) kg rice = Rs. (200 – x)

    ⇒ 15% profit of x + 5% loss of (200 – x) = 10% profit of 200

    ⇒ 15x/100 - 10 + 5x/100 = 20

    ⇒ 20x/100 = 30

    ⇒ x = 150 kg

    ⇒ 200 – x = 200 – 150 = 50 kg

    ∴ Quantity sold at 15% profit = 150 kg and the quantity sold at 5% loss = 50 kg

    माना 15% लाभ पर बेची गई मात्रा x नकलोग्राम है। तो 5% हानन पर बेची गई मात्रा (200 –x) नकलोग्राम होगी।

    हम मान लेते हैं नक चािल का मूल्य 1रू. प्रनत नकलोग्राम है

    तो x नकलोग्राम चािल का मूल्य = x रू. और (200 – x) नकलोग्राम चािल का मूल्य = (200 – x) रू.

    ⇒ x का 15% लाभ + (200 – x) की हानन का 5% = 200 का 10% लाभ

    ⇒ 15x/100 - 10 + 5x/100 = 20

    ⇒ 20x/100 = 30

    ⇒ x = 150 नकलोग्राम

    ⇒ 200 – x = 200 – 150 = 50 नकलोग्राम

    ∴ 15% लाभ पर बेची गई मात्रा = 150 नकलोग्राम और 5% हानन पर बेची गई मात्रा = 50 नकलोग्राम

    Q32.

    https://testbook.com/passfile:///C:/Users/Administrator/Downloads/www.bit.ly/bank-live

  • 29 | P a g e

    Ans: 4

    Solution:

    Based on given data,

    Work done by 3 girls in 1 day = 1/8

    Work done by 3 boys in 1 day = 1/9

    Work done by 7 men in 1 day = 1/3

    Work done by 6 women in 1 day = 1/4

    Assuming all girls are equally efficient,

    Work done by 1 girl in 1 day = 1/3 × 1/8 = 1/24 ---(1)

    Assuming all boys are equally efficient,

    Work done by 1 boy in 1 day = 1/3 × 1/9 = 1/27 ---(2)

    Assuming all men are equally efficient,

    Work done by 1 man in 1 day = 1/7 × 1/3 = 1/21 ---(3)

    Assuming all women are equally efficient,

    Work done by 1 woman in 1 day = 1/6 × 1/4 = 1/24 ---(4)

    From 1,2,3,4,

    1/27 < 1/24 < 1/21

    Thus the least efficiency is 1/27 and the greatest efficiency is 1/21.

    Thus, men are the most efficient.

    दी गई जानकारी के आिार पर,

    1 नदन में तीन लडनकिो ंद्वारा नकिा गिा कािय = 1/8

    1 नदन में तीन लडको ंद्वारा नकिा गिा कािय = 1/9

    1 नदन में सात पुरुषो ंद्वारा नकिा गिा कािय = 1/3

    1 नदन में छः मनहलाओ ंद्वारा नकिा गिा कािय = 1/4

    https://testbook.com/passhttps://play.google.com/store/apps/details?id=com.testbook.tbapp&hl=en

  • 30 | P a g e

    िह मानते हुए नक सभी लडनकिां समान रूप से कुशल हैं,

    1 नदन में 1 लडकी द्वारा नकिा गिा कािय = 1/3 × 1/8 = 1/24 ---(1)

    िह मानते हुए नक सभी लडके समान रूप से कुशल हैं,

    1 नदन में 1 लडके द्वारा नकिा गिा कािय = 1/3 × 1/9 = 1/27 ---(2)

    िह मानते हुए नक सभी पुरुष समान रूप से कुशल हैं,

    1 नदन में 1 पुरुष द्वारा नकिा गिा कािय = 1/7 × 1/3 = 1/21 ---(3)

    िह मानते हुए नक सभी मनहलाएं समान रूप से कुशल हैं,

    1 नदन में 1 मनहला द्वारा नकिा गिा कािय = 1/6 × 1/4 = 1/24 ---(4)

    1,2,3,4 से,

    1/27 < 1/24 < 1/21

    इस प्रकार नू्यनतम कुशलता 1/27 है और अनिकतम कुशलता 1/21 है।

    इस प्रकार, पुरुष सबसे ज्यादा कुशल हैं|

    Q33.

    Ans: 2

    Solution:

    Let marked price be Rs. 100

    ⇒ Commision = 10% of 100 = 10

    ⇒ SP = 90

    ⇒ CP = 90/125 × 100 = Rs. 72

    ⇒ New commission = Rs. 20

    New SP = Rs. 80

    ⇒ Gain percentage = (8 × 100)/72 = 11.1%

    https://testbook.com/passfile:///C:/Users/Administrator/Downloads/www.bit.ly/bank-live

  • 31 | P a g e

    ∴ Gain percentage = 11.1%

    माना नक अंनकत मूल्य 100 रु है

    ⇒ कमीशन = 100 का 10% = 10

    ⇒ निक्रि मूल्य = 90

    ⇒ क्रि मूल्य = 90/125 × 100 = 72 रु

    ⇒ निा कमीशन = 20 रु

    निा निक्रि मूल्य = 80 रु

    ⇒ लाभ प्रनतशत = (8 × 100) / 72 = 11.1%

    ∴ लाभ प्रनतशत = 11.1%

    Q34.

    Ans: 1

    Solution:

    Let, Cost Price of the first table = Rs. y

    ∴ Cost price of the other table = Rs. (1800 - y) [∵ Purchase price of two tables = Rs.1800]

    According the problem,

    Selling price of the first table = Rs. y × 4/5

    Selling price of the second table = Rs. ((1800 - y) × 5) /4

    ∴ 4𝑦

    5+

    (1800−𝑦)×5

    4 [∵ He made an overall gain of Rs. 90]

    https://testbook.com/passhttps://play.google.com/store/apps/details?id=com.testbook.tbapp&hl=en

  • 32 | P a g e

    Or, 4𝑦

    5−

    5𝑦

    4 = 1890 - 2250

    Or, (16y - 25y) /20 = - 360

    Or, - 9y = - 360 × 20

    Or, y = 7200/9

    ⇒ y = 800

    ∴ Cost of the first table = Rs. 800

    Cost of the other table = Rs. (1800 - 800) = Rs. 1000

    ∴ Cost of the lesser valued chair = Rs. 800

    माना प्रथम मेज़ का क्रि मूल्य = y रुपए

    ∴ दूसरी मेज़ का क्रि मूल्य = (1800 - y) रुपए [∵ दोनो ंमेज़ो ंका क्रि मूल्य = 1800 रुपए]

    प्रश्न के अनुसार,

    प्रथम मेज़ का निक्रि मूल्य = y × 4/5 रुपए

    दूसरी मेज़ का निक्रि मूल्य = ((1800 - y) × 5) /4 रुपए

    ∴ 4𝑦

    5+

    (1800−𝑦)×5

    4 [∵ उसे कुल 90 रुपए का लाभ हुआ है]

    िा, 4𝑦

    5−

    5𝑦

    4 = 1890 - 2250

    िा, (16y - 25y) /20 = - 360

    िा, - 9y = - 360 × 20

    िा, y = 7200/9

    ⇒ y = 800

    ∴ प्रथम मेज़ का क्रि मूल्य = 800 रुपए

    दूसरी मेज़ का क्रि मूल्य = (1800 - 800) = 1000 रुपए

    ∴ कम मूल्य िाली मेज़ का क्रि मूल्य = 800 रुपए

    https://testbook.com/passfile:///C:/Users/Administrator/Downloads/www.bit.ly/bank-live

  • 33 | P a g e

    Q35.

    Ans: 2

    Solution:

    Let us assume money invested on scheme X, Y and Z be Rs. x, y and z respectively

    Also assuming interest gain by Dharmesh from each scheme be α

    We know the formula for simple Interest,

    SI = (P × R × T)/100

    Where,

    SI = Simple Interest

    P = Principle

    R = Rate of Interest

    t = Time period

    ∴ Interest gain from scheme X = (x × 4 × 7)/100 = α

    ⇒ x = 100α/28

    Similarly Interest gain from scheme Y = (y × 6 × 4)/100 = α

    ⇒ y = 100α/24

    And Interest gain from scheme Z = (z × 7 × 3)/100 = α

    ⇒ z = 100α/21

    ∴ x : y : z = 100α/28 : 100α/24 : 100α/21

    We know that,

    A : B : C = nA : nB : nC

    https://testbook.com/passhttps://play.google.com/store/apps/details?id=com.testbook.tbapp&hl=en

  • 34 | P a g e

    Where n is a constant

    ∴ By multiplying Right hand side ratio by 42/25 we get,

    ⇒ x : y : z = 6 : 7 : 8

    माना नक िोजना X, Y और Z में ननिेश की गिी रानश क्रमशः x, y और z रूपिे है

    साथ ही माना नक िमेश द्वारा प्रते्यक िोजना से अनजयत ब्याज α है

    हम जानते हैं नक सािारण ब्याज का सूत्र,

    SI = (P × R × T)/100

    जहााँ,

    SI = सािारण ब्याज

    P = मूलिन

    R = ब्याज की दर

    t = समिािनि

    ∴ िोजना X से अनजयत ब्याज = (x × 4 × 7)/100 = α

    ⇒ x = 100α/28

    उसीप्रकार िोजना Y से अनजयत ब्याज = (y × 6 × 4)/100 = α

    ⇒ y = 100α/24

    और िोजना Z से अनजयत ब्याज = (z × 7 × 3)/100 = α

    ⇒ z = 100α/21

    ∴ x : y : z = 100α/28 : 100α/24 : 100α/21

    हम जानते हैं नक,

    https://testbook.com/passfile:///C:/Users/Administrator/Downloads/www.bit.ly/bank-live

  • 35 | P a g e

    A : B : C = nA : nB : nC

    जहााँ n एक क्तस्थरांक है

    ∴ दाएाँ पक्ष के अनुपात को 42/25 से गुणा करने पर हमें प्राप्त होता है,

    ⇒ x : y : z = 6 : 7 : 8

    https://testbook.com/passhttps://play.google.com/store/apps/details?id=com.testbook.tbapp&hl=en